Đến nội dung

Nguyen Minh Hai nội dung

Có 652 mục bởi Nguyen Minh Hai (Tìm giới hạn từ 25-05-2020)



Sắp theo                Sắp xếp  

#523981 TOPIC VỀ CÁC BÀI HÌNH HỌC LỚP 7,8

Đã gửi bởi Nguyen Minh Hai on 11-09-2014 - 22:35 trong Hình học

Bài 1: Cho hình thoi ABCD có $\widehat{BAD}=120^{\circ}$ , điểm M thuộc cạnh AB , DM cắt BC tại N, CM cắt AN tại E. Chứng minh rằng:

            a) $\Delta AMD \sim \Delta CDN$

            b) $\Delta AME \sim \Delta CMB$

Bài 2: Cho $\Delta ABC$ có $AB > AC$

            a) Vẽ đường cao BM, CN của tam giác. CMR:

                   $\Delta ABM \sim \Delta ACN$    và       $\widehat{AMN}=\widehat{ABC}$

            b) Lấy điểm K thuộc AB sao cho BK = AC. E là trung điểm BC, F là trung điểm AK. CMR:

                  EF // tia phân giác Ax của $\widehat{BAC}$




#633449 Inequalities From 2016 Mathematical Olympiads

Đã gửi bởi Nguyen Minh Hai on 16-05-2016 - 14:38 trong Bất đẳng thức - Cực trị

 

Mình tổng hợp lại một số bài chưa có lời giải, mọi người cùng suy nghĩ thử nhé :)

 

Bài 28 (Russia). Cho bốn số thực dương $a, b, c, d$ thỏa mãn điều kiện $a+b+c+d=3.$ Chứng minh rằng $$\frac{1}{a^2}+\frac{1}{b^2}+\frac{1}{c^2}+\frac{1}{d^2}\le\frac{1}{a^2b^2c^2d^2},$$


$$\frac{1}{a^3}+\frac{1}{b^3}+\frac{1}{c^3}+\frac{1}{d^3}\le\frac{1}{a^3b^3c^3d^3}.$$

 

 

Lời giải 

 $(a)$       Giả sử $a \geqslant b \geqslant c \geqslant d$, BĐT cần chứng minh tương đương với

$a^{2}b^{2}c^{2}+a^{2}b^{2}d^{2}+a^{2}c^{2}d^{2}+b^{2}c^{2}d^{2} \leqslant 1$

Ta đưa BĐT về dạng đồng bậc

$a^{2}b^{2}c^{2}+a^{2}b^{2}d^{2}+a^{2}c^{2}d^{2}+b^{2}c^{2}d^{2} \leqslant \frac{1}{3^6}(a+b+c)^6$

Do $a \geqslant b \geqslant c \geqslant d$ nên ta

\[\begin{aligned} a^{2}b^{2}c^{2}+a^{2}b^{2}d^{2}+a^{2}c^{2}d^{2}+b^{2}c^{2}d^{2} &\leqslant a^{2}b^{2}c^{2}+a^{2}b^{2}d^{2}+a^{2}b^{2}cd+a^{2}b^{2}cd \\ &= a^{2}b^{2}(c+d)^{2} \\ &\leqslant \frac{1}{3^{6}}(a+b+c+d)^{6} \\ &=1\end{aligned}\]

Bài toán được chứng minh. $\square$ 

 $(b)$       Giả sử $a \geqslant b \geqslant c \geqslant d$, BĐT cần chứng minh tương đương với

$a^{3}b^{3}c^{3}+a^{3}b^{3}d^{3}+a^{3}c^{3}d^{3}+b^{3}c^{3}d^{3} \leqslant 1$

Ta đưa BĐT về dạng đồng bậc

$a^{3}b^{3}c^{3}+a^{3}b^{3}d^{3}+a^{3}c^{3}d^{3}+b^{3}c^{3}d^{3} \leqslant \frac{1}{3^9}(a+b+c)^9$

Theo BĐT $AM - GM$ ta

$ \frac{1}{3^9}(a+b+c+d)^9 \geqslant \frac{1}{3^9}\left(3\sqrt[3]{ab(c+d)} \right)^9 = a^3b^3(c+d)^3 $

Do đó ta chỉ cần chứng minh $a^{3}b^{3}c^{3}+a^{3}b^{3}d^{3}+a^{3}c^{3}d^{3}+b^{3}c^{3}d^{3} \leqslant a^3b^3(c+d)^3$ $\Leftrightarrow c^3d^3(a^3+b^3) \leqslant 3a^3b^3cd(c+d)$ $\Leftrightarrow c^2d^2(a^3+b^3) \leqslant 3a^3b^3(c+d)$

BĐT này luôn đúng do $a \geqslant b \geqslant c \geqslant d$

Bài toán được chứng minh. $\square$




#633413 Inequalities From 2016 Mathematical Olympiads

Đã gửi bởi Nguyen Minh Hai on 16-05-2016 - 01:25 trong Bất đẳng thức - Cực trị

 

Mình tổng hợp lại một số bài chưa có lời giải, mọi người cùng suy nghĩ thử nhé :)

 

 

Bài 27 (Iran Second Round). Cho ba số thực $0<a \leqslant b \leqslant c.$ Chứng minh rằng
$$\frac{(c-a)^2}{6c}\leq \frac{a+b+c}{3}-\frac{3}{\frac{1}{a}+\frac{1}{b}+\frac{1}{c}}.$$

 

Hình gửi kèm

  • iran second round.jpg



#633411 Inequalities From 2016 Mathematical Olympiads

Đã gửi bởi Nguyen Minh Hai on 16-05-2016 - 00:44 trong Bất đẳng thức - Cực trị

Một cách tiếp cận khác cho Bài 30

Bài 30 (EMMO). Cho ba số thực $a,b,c$ thỏa mãn điều kiện $a^2+b^2+c^2=4+abc.$ Chứng minh rằng $$(a^2+2)(b^2+2)(c^2+2)\ge 9+6(ab+bc+ca).$$
 

Đặt $t = \sqrt[3]{abc}$ thì từ giả thiết bài toán ta

$4+abc = a^{2}+b^{2}+c^{2} \geqslant 3\sqrt[3]{a^{2}b^{2}c^{2}}$

$\Leftrightarrow 4+t^3 \geqslant 3t^2  \Leftrightarrow  (t+1)(t+2)^2 \geqslant 0$

 

Từ đó suy ra $abc \geqslant -1$, kết hợp với giả thiết suy ra $a^{2}+b^{2}+c^{2} \geqslant 3$.

 

Ta BĐT quen thuộc sau $(a^{2}+2)(b^{2}+2)(c^{2}+2) \geqslant 3(a+b+c)^{2}   (1)$

 

Thật vậy, theo BĐT $Cauchy - Schwarz$ thì ta

 

$$(a+b+c)^{2} \leqslant (a^{2}+2)\left[1+\frac{(b+c)^2}{2}\right]$$

 

Do đó để chứng minh $(1)$ ta chỉ cần chứng minh

$$(b^{2}+2)(c^{2}+2) \geqslant 3\left[1+\frac{(b+c)^2}{2}\right]$$

 

$$\Leftrightarrow \frac{1}{2}(b-c)^2+(bc-1)^2 \geqslant 0$$

 

Do đó $(1)$ được chứng minh.

 

 Áp dụng $(1)$ thì ta quy bài toán về chứng minh

 

$$(a+b+c)^{2} \geqslant 3+2(ab+bc+ca)$$

 

$$\Leftrightarrow a^{2}+b^{2}+c^{2} \geqslant 3$$ 

 

BĐT này luôn đúng. Do đó bài toán được chứng minh 

 Xảy ra đẳng thức khi $a=b=c=-1$. $\square$




#633352 Inequalities From 2016 Mathematical Olympiads

Đã gửi bởi Nguyen Minh Hai on 15-05-2016 - 21:23 trong Bất đẳng thức - Cực trị

 

Mình tổng hợp lại một số bài chưa có lời giải, mọi người cùng suy nghĩ thử nhé :)

 

 

Bài 23 (Romania JBMO TST 2016). Với $m,n$ là hai số tự nhiên và ba số thực $x,y,z$ thuộc $[0,1].$ Chứng minh rằng

\[0 \leqslant x^{m+n}+y^{m+n}+z^{m+n}-x^my^n-y^mz^n-z^mx^n \leqslant 1.\]
Đẳng thức xảy ra khi nào ?

 

$(a)$      Chứng minh $ x^{m+n}+y^{m+n}+z^{m+n} \geqslant x^my^n+y^mz^n+z^mx^n$

Gọi $(a,b,c)$ hoán vị của $(x,y,z)$ sao cho $a \geqslant b \geqslant c$

 

Khi đó ta $a^m \geqslant b^m \geqslant c^m$ $a^n \geqslant b^n \geqslant c^n$.

 

Do đó theo BĐT Hoán vị ta

$$a^ma^n+b^mb^n+c^mc^n \geqslant a^mb^n+b^mc^n+c^ma^n$$

$$a^ma^n+b^mb^n+c^mc^n \geqslant a^mc^n+b^ma^n+c^ma^n$$

ta

$$a^ma^n+b^mb^n+c^mc^n=x^{m+n}+y^{m+n}+z^{m+n}$$

$$x^my^n+y^mz^n+z^mx^n \in \lbrace a^mb^n+b^mc^n+c^ma^n , a^mc^n+b^ma^n+c^ma^n \rbrace$$

Do đó vế trái của BĐT được chứng minh. Xảy ra đẳng thức khi $x=y=z$

$(b)$         Chứng minh

$$x^my^n+y^mz^n+z^mx^n+1 \geqslant x^{m+n}+y^{m+n}+z^{m+n}$$

 

Giả sử $x=max\lbrace x,y,z \rbrace$ thì BĐT được viết lại thành

 

$$(1-x^{m+n})+y^n(x^m-y^m)+z^m(x^n-z^n)+y^mz^n \geqslant 0$$

 

BĐT cuối luôn đúng do $x=max\lbrace x,y,z \rbrace$ $x,y,z \in [0;1]$.

 

Vậy vế phải BĐT được chứng minh. Xảy ra đẳng thức khi $x=1,y=z=0$ các hoán vị. $\square$




#570305 Bánh canh chém gió về kì thi IMO 2015

Đã gửi bởi Nguyen Minh Hai on 07-07-2015 - 00:17 trong Thi HSG Quốc gia và Quốc tế

Theo em biết thì đả có 2 anh không học trường chuyên và từng được đi thi IMO rồi 




#571047 Bánh canh chém gió về kì thi IMO 2015

Đã gửi bởi Nguyen Minh Hai on 10-07-2015 - 15:23 trong Thi HSG Quốc gia và Quốc tế

Đề thi ngày hôm nay đây thì phải IMO 2015 Day 1.jpg




#571038 Bánh canh chém gió về kì thi IMO 2015

Đã gửi bởi Nguyen Minh Hai on 10-07-2015 - 15:11 trong Thi HSG Quốc gia và Quốc tế

Sao người ta không đây thời gian lên thi vào buổi chiều nhỉ? Lại thi vào giữa trưa.




#520408 [TOPIC] Luyện tập biến đổi căn thức

Đã gửi bởi Nguyen Minh Hai on 19-08-2014 - 22:01 trong Đại số

Các anh chị giải giùm em bài này với ạ! 
Cho biểu thức:  
$A=\frac{\sqrt{x-2+\sqrt{2x-5}}}{x\sqrt{2}-3\sqrt{2}}$

a) Rút gọn biểu thức

b) Tính A khi $x=4\sqrt{2}$
 




#537796 [TOPIC] Luyện tập biến đổi căn thức

Đã gửi bởi Nguyen Minh Hai on 13-12-2014 - 23:47 trong Đại số

 

4/Tính : $Q= \frac{\sqrt[3]{\frac{1}{9}} - \sqrt[3]{\frac{2}{9}}+ \sqrt[3]{\frac{4}{9}}}{\sqrt[3]{\sqrt[3]{2}} -1}$

$Q=\frac{(\sqrt[3]{\frac{1}{3}})^2-\sqrt[3]{\frac{1}{3}}.\sqrt[3]{\frac{2}{3}}+(\sqrt[3]{\frac{2}{3}})^2}{\sqrt[9]{2}-1}=\frac{\frac{2}{3}+\frac{1}{3}}{(\sqrt[3]{\frac{2}{3}}+\sqrt[3]{\frac{1}{3}})(\sqrt[9]{2}-1)}=\frac{\sqrt[3]{3}}{(\sqrt[3]{2}+1)(\sqrt[9]{2}-1)}$

Ai rút gọn tiếp xem được k! :D




#552300 $\boxed {\textbf{TOPIC}}$ Ôn thi VIOL...

Đã gửi bởi Nguyen Minh Hai on 07-04-2015 - 23:03 trong Cuộc thi VIOlympic (Cuộc thi do Bộ giáo dục và đào tạo tổ chức)

Bài 37:

Cho $(O)$ nội tiếp tứ giác$ABCD$. Gọi $M,N$ lần lượt là các tiếp điểm của các cạnh $AD,DC$ với $(O)$. Đường chéo $AC$ cắt $(O)$ tại hai điểm $E,F$ ($E$ nằm giữa $A$ và $F$) và cắt $MN$ tại $I$. Biết $AE=2,EF=6,FC=3$ và tỉ số $\frac{CI}{AI}=\sqrt{\frac{a}{b}}$. Tính $a-b$




#551175 $\boxed {\textbf{TOPIC}}$ Ôn thi VIOL...

Đã gửi bởi Nguyen Minh Hai on 03-04-2015 - 19:29 trong Cuộc thi VIOlympic (Cuộc thi do Bộ giáo dục và đào tạo tổ chức)

Bài 30: 

$S_{CID}$ đạt $max$ khi $ABCD$ là hình thang cân. dễ dàng tính được $Max(S_{CID})=4$




#551172 $\boxed {\textbf{TOPIC}}$ Ôn thi VIOL...

Đã gửi bởi Nguyen Minh Hai on 03-04-2015 - 19:23 trong Cuộc thi VIOlympic (Cuộc thi do Bộ giáo dục và đào tạo tổ chức)

Bài 31: - Kết quả là 37,4.

Kẻ các đường cao $AH,BK$

Đặt $CK=x(x>0)$ $\Rightarrow DH=9-x$

$AH=BK=\sqrt{49-x^2}$

$\widehat{ADH}+\widehat{BCK}=90^{\circ}$

$\Leftrightarrow tan^{-1}(\frac{\sqrt{49-x^2}}{9-x})+cos^{-1}(\frac{x}{7})=90$

Giải PT trên bằng máy Casio ta được: $x=\frac{49}{9}$

Do đó $S\approx 37,4$

P/s: Mình là dân Casio :D 




#523566 QUỸ TÍCH HÌNH HỌC

Đã gửi bởi Nguyen Minh Hai on 08-09-2014 - 22:45 trong Chuyên đề toán THCS

Cho tam giác ABC cố định. Xét các hình chữ nhật có 2 đỉnh trên cạnh BC, hai đỉnh còn lại thuộc hay cạnh kia của tam giác. Tìm quỹ tích tâm của các hình chữ nhật này

Mọi người cùng giải nhanh giùm mình nao! : :icon6:




#583692 TOPIC Tổ hợp-Xác suất

Đã gửi bởi Nguyen Minh Hai on 21-08-2015 - 19:29 trong Tổ hợp - Xác suất và thống kê - Số phức

 

$\boxed{2}$Một văn phòng cần chọn mua một tờ nhật báo mỗi ngày.Có 4 loại nhật báo.Hỏi có mấy cách chọn mua báo cho một tuần gồm 6 ngày làm việc?

 

- Ngày thứ nhất có 4 cách chọn mua nhật báo.

- Ngày thứ 2 cũng có 4 cách chọn mua nhật báo.

...

- Tương tự thì ngày thứ 6 cũng có 4 cách chọn mua nhật báo.

Áp dụng quy tắc nhân suy ra tổng số cách chọn mua nhật báo trong 6 ngày sẽ là:    $4^6=4096$ cách.




#571041 CÁC CÂU HỎI SUY LUẬN VỀ VẤN ĐỀ TRINH THÁM

Đã gửi bởi Nguyen Minh Hai on 10-07-2015 - 15:14 trong IQ và Toán thông minh

Câu 8: 1 hôm có 1 thám tử nổi tiếng đi Hawai nghỉ mát, ông đi bằng 1 chiếc thuyền du lịch hạng sang. Trên thuyền có 5 người khác kể cả vị thám tử. Đang đi thì thuyền gặp bảo nên mọi người mau chóng về phòng. Sau 25 phút bão tan thì có 1 phụ nữ sang trọng tìm gặp thám tử và nói rằng mình bị cướp. Bà ta kể lúc đó bà đang ở trong phòng thì có người gọi cửa sau đó bà bị chụp khăn vào mặt và ngất xỉu. Thám tử liền gọi mọi người tới lấy lời khai và đây là lời khai của họ:

1- Tôi về phòng uống rượu và gọi điện cho bạn khoảng 30 phút

2- Tôi phụ người trong bếp xếp đồ vì bão nên nhiều đồ ngã vỡ 

3- Tôi chỉ ở trong phòng lên facebook

4- Tôi viết thư cho mẹ ông có thể xem bức thư chữ viết rất rõ ràng tôi phải nắn nót 30 phút đó

Vậy ai là thủ phạm

Ông có lời khai thứ 4 là thủ phạm. 




#639955 VMF's Marathon Bất Đẳng Thức Olympic

Đã gửi bởi Nguyen Minh Hai on 13-06-2016 - 00:20 trong Bất đẳng thức và cực trị

 

Bài toán 31 (Võ Quốc Bá Cẩn). Cho các số thực dương $a,b,c$ thỏa mãn $a+b+c=5$. Chứng minh
$$a^2b+c^2a+2abc \leq 20.$$

 

Lời giải bài 31. Đặt $f(a,b,c)=a^2b+c^2a+2abc$

Ta xét $2$ trường hợp.

Nếu $a \geqslant c$, ta sẽ chứng minh 

$$f(a,b,c) \leqslant f(a,b,a)$$

$$\Leftrightarrow (a-c)(a+c+2b) \geqslant 0$$

 

Nếu $a \leqslant c$, ta sẽ chứng minh 

$$f(a,b,c) \leqslant f(c,b,c)$$

$$\Leftrightarrow b(c^2-a^2)+c^2(c-a)+2bc(c-a) \geqslant 0$$

 

Do đó ta chỉ cần chứng minh BĐT trong trường hợp $a=c=\frac{5-b}{2}$

$$\Leftrightarrow \left ( \frac{5-b}{2}\right )^2b+\left ( \frac{5-b}{2} \right)^3+2b\left (\frac{5-b}{2} \right )^2 \leqslant 20$$

$$\Leftrightarrow  b^3-9b^2+15b-7 \leqslant 0$$

$$\Leftrightarrow (b-7)(b-1)^{2} \leqslant 0$$

BĐT cuối luôn đúng do $b<5$

Bài toán được chứng minh. Xảy ra đẳng thức khi $a=c=2,b=1$

 

Bài toán 32.  (Vasile Cirtoaje) Cho các số thực dương $a,b,c,d$. Chứng minh rằng 

$$2(a^3+1)(b^3+1)(c^3+1)(d^3+1) \geqslant (1+abcd)(1+a^2)(1+b^2)(1+c^2)(1+d^2)$$




#640894 VMF's Marathon Bất Đẳng Thức Olympic

Đã gửi bởi Nguyen Minh Hai on 17-06-2016 - 17:32 trong Bất đẳng thức và cực trị

Vế phải là mũ $3$ mới phải  :mellow:

Mình nhầm :3 

 

P/s: Long Vá xóa bài giùm anh  :wacko:




#640882 VMF's Marathon Bất Đẳng Thức Olympic

Đã gửi bởi Nguyen Minh Hai on 17-06-2016 - 16:24 trong Bất đẳng thức và cực trị

Bài 35:(VQBC) Cho $a,b,c$ là các số thực ko âm, chứng minh rằng : 

$$2(\frac{a}b{+\frac{b}c{+\frac{c}{a}}})+1\geq \frac{21(a^{2}+b^{2}+c^{2})}{(a+b+c)^{2}}$$

 

Lời giải bài 35. (Cách khác)

 

Sử dụng bổ đề 

$$(x+y+z)^3 \geqslant \frac{27}{4}(a^2b+b^2c+c^2a+abc)$$

ta có

$$left(\frac{a}{b}+\frac{b}{c}+\frac{c}{a} \right )^3 \geqslant \frac{27}{4}\left( \frac{a^3+b^3+c^3}{abc}+1 \right)$$

 

Do đó BĐT cần chứng minh trở thành 

 

$$54\left( \frac{a^3+b^3+c^3}{abc}+1 \right) \geqslant \left( \frac{21(a^2+b^2+c^2)}{(a+b+c)^2}-1\right)^2$$

 

$$\Leftrightarrow 54\left( \frac{a^3+b^3+c^3}{abc}-3 \right) \geqslant \left( \frac{441(a^2+b^2+c^2)^2}{(a+b+c)^4}-49 \right)-\left(\frac{42(a^2+b^2+c^2}{(a+b+c)^2}-14\right)$$

 

$$\Leftrightarrow \frac{54(a+b+c)(\sum a^2-\sum ab)}{abc} \geqslant \frac{14(\sum a^2-\sum ab)\left[21\sum a^2+7(a+b+c)^2\right]}{(a+b+c)^4}-\frac{28(\sum a^2-\sum ab)}{(a+b+c)^2}$$

 

$$\Leftghtarrow \left(\sum a^2-\sum ab \right)\left( \frac{27(a+b+c)}{abc}+\frac{14}{(a+b+c)^2}-\frac{147\sum a^2+49(a+b+c)^2}{(a+b+c)^4} \right)$$

 

Do đó ta chỉ cần chứng minh 

 

$$\frac{27(a+b+c)}{abc}+\frac{14}{(a+b+c)^2}-\frac{147\sum a^2+49(a+b+c)^2}{(a+b+c)^4} \geqslant 0$$

 

Chuẩn hóa $a+b+c=1$ thì ta cần chứng minh 

$$\frac{27}{abc}+14 \geqslant 147(a^2+b^2+c^2)+49$$

$$\Leftrightarrow 147abc(a^2+b^2+c^2)+35abc \leqslant 27$$

 

BĐT cuối luôn đúng do 

$$abc \leqslant \frac{1}{27}$$

và 

$$a^2+b^2+c^2 \leqslant (a+b+c)^2 =1$$

 

BĐT được chứng minh. Xảy ra đẳng thức khi $a=b=c$




#568182 Topic tổng hợp một số bất đẳng thức trong kì thi MO các nước

Đã gửi bởi Nguyen Minh Hai on 25-06-2015 - 21:41 trong Bất đẳng thức - Cực trị

Bài 153: (Việt Nam TST 2005) 

Cho $a,b,c>0$. Chứng minh: $\left ( \frac{a}{a+b} \right )^3+\left ( \frac{b}{b+c} \right )^3+\left ( \frac{c}{c+a} \right )^3 \geq \frac{3}{8}$




#566805 Topic tổng hợp một số bất đẳng thức trong kì thi MO các nước

Đã gửi bởi Nguyen Minh Hai on 19-06-2015 - 09:22 trong Bất đẳng thức - Cực trị

 

Bài 117 (CĐTMO 2001) : Xét các số thực dương $a,b,c$ thỏa mãn điều kiện 

                                                                          $21ab+2bc+8ca\leq 12$
                                        Tìm giá trị nhỏ nhất của biểu thức $P=\frac{1}{a}+\frac{2}{b}+\frac{3}{c}.$ 

 

Spoiler


 

Bài toán này đã có khá nhiều cách giải trên diễn đàn

http://diendantoanho...-frac2b-frac3c/

http://diendantoanho...c3cgeq-frac152/

Thêm một cách mà mình vừa nghĩ ra  :icon10: 

Từ điểm rơi của bài toán là $(a,b,c)=\left ( \frac{1}{3};\frac{4}{5};\frac{3}{2} \right )$

$\Rightarrow 36a=15b=8c=12$

Do đó ta đặt: $\left\{\begin{matrix} 36a=x & & & \\ 15b=y & & & \\ 8c=z & & & \end{matrix}\right.$

Bài toán trở thành: 

Cho $x,y,z>0$ thõa mãn: $\frac{7xy}{180}+\frac{yz}{60}+\frac{zx}{36} \leq 12$.TÌm GTNN của:

$P=\frac{36}{x}+\frac{30}{y}+\frac{24}{z}$

Từ giả thiết ta có: 

$2160 \geq 7xy+3yz+5zx \geq 15\sqrt[15]{x^7y^7.y^3z^3.z^5x^5}$

$\Rightarrow x^6y^5z^4 \leq 12^5$            (Theo $AM-GM$)

Do đó: $P=6(\frac{6}{x}+\frac{5}{y}+\frac{4}{z})\geq 6.15\sqrt[15]{\frac{1}{x^6y^5z^4}} \geq 90.\sqrt[15]{\frac{1}{12^{15}}}=\frac{15}{2}$

$\Rightarrow GTNN_{P}=\frac{15}{2}$ khi $(a,b,c)=\left ( \frac{1}{3};\frac{4}{5};\frac{3}{2} \right )$

[spoiler] Đề thi vòng 16 Vio 9 năm nay cũng ra bài này [\spoiler]




#566811 Topic tổng hợp một số bất đẳng thức trong kì thi MO các nước

Đã gửi bởi Nguyen Minh Hai on 19-06-2015 - 09:48 trong Bất đẳng thức - Cực trị

 

Bài 116 (CĐTMO 2005) : Chứng minh rằng
                                                                 $\frac{a^{3}}{(b+c)^{3}}+\frac{b^{3}}{(c+a)^{3}}+\frac{c^{3}}{(a+b)^{3}}$

                                        trong đó $a,b,c$ là các số dương.

 

 

Có vẽ đề bị thiếu thì phải?  :closedeyes:




#566962 Topic tổng hợp một số bất đẳng thức trong kì thi MO các nước

Đã gửi bởi Nguyen Minh Hai on 19-06-2015 - 21:01 trong Bất đẳng thức - Cực trị

 

Bài 116 (CĐTMO 2005) : Chứng minh rằng
                                                                 $\frac{a^{3}}{(b+c)^{3}}+\frac{b^{3}}{(c+a)^{3}}+\frac{c^{3}}{(a+b)^{3}}$ $\geq \frac{3}{8}$

                                        trong đó $a,b,c$ là các số dương.

 

 

Ta có BĐT quen thuộc: $\frac{x^3+y^3+z^3}{3}\geq \left ( \frac{x+y+z}{3} \right )^3$ (Chứng minh bằng $AM-GM$)

Áp dụng BĐT trên ta được:

$VT \geq \frac{1}{9}\left (\frac{a}{b+c}+\frac{b}{c+a}+\frac{c}{a+b} \right )^3\geq \frac{1}{9}.\frac{27}{8}=\frac{3}{8}$




#567103 Topic tổng hợp một số bất đẳng thức trong kì thi MO các nước

Đã gửi bởi Nguyen Minh Hai on 20-06-2015 - 15:37 trong Bất đẳng thức - Cực trị

Bài 123:(Mở rộng của bài 118)(tự sáng tác) : Cho a,b,c>0. Tìm hằng số k tốt nhất sao cho bđt sau đúng:

 

$\sqrt{\frac{a^3+b^3+1}{k(b^2+c^2)+13bc}}+\sqrt{\frac{b^3+c^3+1}{k(c^2+a^2)+13ca}}+\sqrt{\frac{c^3+a^3+1}{k(a^2+b^2)+13ab}}\geq 1$

Sử dụng BĐT $AM-GM$ cho vế trái ta được:

$\sum \sqrt{\frac{a^3+b^3+1}{k(b^2+c^2)+13bc}} \geq 3.\sqrt[6]{\frac{\prod( a^3+b^3+1)}{\prod(k(b^2+c^2)+13bc) }}$

 

Theo BĐT $Holder$ ta lại có:

$\prod (a^3+b^3+1)\geq(a^2+b^2+1)^3$

 

Và theo $AM-GM$ ta có:

$\prod \left [ k(b^2+c^2)+13bc \right ]\leq\prod \left ( k+\frac{13}{2} \right )(b^2+c^2)$

$\leq \frac{8}{27}.\left ( k+\frac{13}{2} \right )^3(a^2+b^2+c^2)^3$

 

Do đó: $VT \geq 3.\sqrt[6]{\frac{27}{8(k+\frac{13}{2})^3}}=3\sqrt{\frac{3}{2(k+\frac{13}{2})}} \geq 1$

$\Rightarrow k \leq 7$

 

Do đó hằng số $k$ tốt nhất để BĐT đề bài đùng là $k=7$




#567548 Topic tổng hợp một số bất đẳng thức trong kì thi MO các nước

Đã gửi bởi Nguyen Minh Hai on 22-06-2015 - 22:41 trong Bất đẳng thức - Cực trị

Bài 139(Kazakstan MO): Cho a,b,c>0. CMR:

$\sum \frac{1}{a^2+ab+b^2}\geq \frac{9}{(a+b+c)^2}$

BĐT cần chứng minh tương đương với

$\sum \frac{a^2+b^2+c^2+ab+bc+ca}{a^2+ab+b^2} \geq \frac{9(a^2+b^2+c^2+ab+bc+ca)}{(a+b+c)^2}$

 

$\Leftrightarrow \sum \left ( \frac{(a^2+ab+b^2)+c(a+b+c)}{a^2+b^2+c^2} \right ) \geq \frac{9(a^2+b^2+c^2+ab+bc+ca)}{(a+b+c)^2}$

 

$\Leftrightarrow 3+(a+b+c).\sum \frac{c}{a^2+ab+b^2} \geq \frac{9(a^2+b^2+c^2+ab+bc+ca)}{(a+b+c)^2}$

 

Áp dụng $Cauchy-Schwarz$ ta có:

$\sum \frac{c}{a^2+ab+b^2}\geq \frac{(a+b+c)^2}{\sum c(a^2+ab+b^2)}=\frac{a+b+c}{ab+bc+ca}$

 

Do đó ta cần chứng minh:

$3+\frac{(a+b+c)^2}{ab+bc+ca} \geq \frac{9(a^2+b^2+c^2+ab+bc+ca)}{(a+b+c)^2}$

 

$\Leftrightarrow 3+\frac{(a+b+c)^2}{ab+bc+ca}\geq \frac{9[(a+b+c)^2-(ab+bc+ca)]}{(a+b+c)^2}$

 

$\Leftrightarrow \frac{(a+b+c)^2}{ab+bc+ca}+\frac{9(ab+bc+ca)}{(a+b+c)^2} \geq 6$

 

Tuy nhiên BĐT cuối luôn đúng theo $AM-GM$

Do đó bài toán được chứng minh xong!